On se propose de démontrer que la probabilité \(r_n\) que deux entiers pris au hasard dans \(\{1,\dots,n\}\) soient premiers entre-eux vérifie \[r_n=\dfrac{1}{n^2}\sum_{d\geq 1}\mu(d)E\left( \dfrac{n}{d}\right)^2\quad\text{et}\quad \lim_n r_n=\dfrac{6}{\pi^2}\] où l’application (c’est la fonction de möebius) \(\mu\ :\ \mathbb N^\star\to\ \{-1,0,1\}\) est définie par : \[\mu(n)=\begin{cases} &1\qquad\quad\text{ si }n=1,\\ &0\qquad\quad\text{ si }n\text{ possède au moins un facteur carré, }\\ &(-1)^k\quad\text{ si }n=p_1 \dots p_k\text{où les }p_i\text{ sont des nombres premiers distincts.} \end{cases}\] Soient \(p_1,\dots,p_k\) les nombres premiers \(\leq n\) et pour \(1\leq i\leq k\) : \[V_i:=\left\lbrace (a,b)\in\{1,\dots,n\}^2\ :\ p_i\text{ divise }a\text{ et }b\right\rbrace .\]

Montrer que

\[\begin{aligned}\text{card}\left( \bigcup_{i=1}^k V_i\right) &=\sum_{\emptyset\neq I\subset\{1,\dots,n\}}(-1)^{1+\text{card}(I)}\text{card}\left( \bigcap_{i\in I}V_i\right) \\ &=-\sum_{\emptyset\neq I\subset\{1,\dots,n\}} (-1)^{\text{card}(I)}E\left(\dfrac{n}{\prod_{i\in I}p_i}\right)^2\\ &=-\sum_{d=2}^n\mu(d)E\left( \dfrac{n}{d}\right)^2\end{aligned}\] Et en déduire \(r_n\).

Montrer que \(\displaystyle\left\vert r_n-\sum_{d=1}^n \dfrac{\mu(d)}{d^2}\right\vert=0\left( \dfrac{\log(n)}{n}\right).\)

Montrer que \(\displaystyle\sum_{n\geq 1}\dfrac{1}{n^2}\sum_{d\geq 1} \dfrac{\mu(d)}{d^2}=\sum_{i\geq 1}\sum_{l\text{ divise } i}\dfrac{\mu(l)}{i^2}=1.\)

Conclure.


Barre utilisateur

[ID: 2542] [Date de publication: 9 novembre 2022 12:23] [Catégorie(s): Combinatoires et probabilités ] [ Nombre commentaires: 1] [nombre d'éditeurs: 1 ] [Editeur(s): Emmanuel Vieillard-Baron ] [nombre d'auteurs: 1 ] [Auteur(s): Patrice Lassère ]




Solution(s)

Solution(s)

Probabilité que deux entiers soient premiers entre-eux
Par Patrice Lassère le 9 novembre 2022 12:23
  1. Soit donc \(n\geq 1\) et désignons par \(p_1,p_2,\dots,p_k\) les nombres premiers inférieurs ou égaux à \(n\). Avec la formule du crible \[\text{card}\left( \bigcup_{i=1}^kV_i\right) = \sum_{\emptyset\neq I\subset\{1,\dots,n\}}(-1)^{1+\text{card}(I)}\text{card}\left( \bigcap_{i\in I}V_i\right)\] soit \(I\subset\{1,\dots,n\}\) une partie non vide, le nombre de multiples de \(\prod_{i\in I}p_i\) dans \(\{1,\dots,n\}\) est \(E\left( \frac{n}{\prod_{i\in I}p_i}\right)\) soit \[\text{card}\left( \bigcap_{i\in I}V_i\right)=E\left( \dfrac{n}{\prod_{i\in I}p_i}\right)^2\] si bien qu’avec la formule du crible le nombre de couples d’entiers inférieurs ou égaux à \(n\) est \[\begin{aligned}\text{card}\left( \bigcup_{i=1}^kV_i\right) &= \sum_{\emptyset\neq I\subset\{1,\dots,n\}}(-1)^{1+\text{card}(I)}\text{card}\left( \bigcap_{i\in I}V_i\right)\\ &= \sum_{\emptyset\neq I\subset\{1,\dots,n\}}(-1)^{1+\text{card}(I)}E\left( \dfrac{n}{\prod_{i\in I}p_i}\right)^2, \qquad{(\bigstar)} \end{aligned}\] il faut maintenant remarquer que puisque \(\mu\left({\prod_{i\in I}p_i} \right)=(-1)^{\text{card}(I)}\) et \(\mu(l)=0\) pour tout autre entier \(l\in\{2,\dots,n\}\) (ces derniers possèdent un facteur carré): \[\sum_{\emptyset\neq I\subset\{1,\dots,n\}}(-1)^{1+\text{card}(I)}E\left( \dfrac{n}{\prod_{i\in I}p_i}\right)^2=-\sum_{d=2}^n\mu(d)E\left( \dfrac{n}{d}\right)^2\] soit finalement \[\text{card}\left( \bigcup_{i=1}^kV_i\right)=-\sum_{d=2}^n\mu(d)E\left( \dfrac{n}{d}\right)^2.\] Ainsi, le nombre de couples \((a,b)\in\{1,2,\dots,n\}^2\) premiers entre-eux est \[n^2-\text{card}\left( \bigcup_{i=1}^kV_i\right)=n^2+\sum_{d=2}^n\mu(d)E\left( \dfrac{n}{d}\right)^2=\sum_{d=1}^n\mu(d)E\left( \dfrac{n}{d}\right)^2\] et la probabilité cherchée est \[r_n=\dfrac{\text{cas favorables}}{\text{cas possibles}}=\dfrac{1}{n^2} \sum_{d=1}^n\mu(d)E\left( \dfrac{n}{d}\right)^2.\]

  2. Commencons par remarquer que pour \(1\leq d\leq n\) \[\left( \dfrac{n}{d}\right)^2 \geq E\left(\dfrac{n}{d}\right)^2 > \left( \dfrac{n}{d}-1\right)^2\] qui implique \[0\geq \dfrac{1}{n^2}E\left(\dfrac{n}{d}\right)^2-\dfrac{1}{d^2}\geq \dfrac{1}{n^2}-\dfrac{2}{nd}\] de sorte que \[\begin{aligned} \left\vert r_n-\sum_{d=1}^n\dfrac{\mu(d)}{d^2}\right\vert &= \left\vert\sum_{d=1}^n\dfrac{\mu(d)}{n^2}E\left( \dfrac{n}{d}\right)^2-\sum_{d=1}^n\dfrac{\mu(d)}{d^2}\right\vert\\ &=\left\vert\sum_{d=1}^n\mu(d)\left[ \dfrac{1}{n^2}E\left(\dfrac{n}{d}\right)^2-\dfrac{1}{d^2}\right]\right\vert \\ &\leq \sum_{d=1}^n \vert\mu(d)\vert\left( \dfrac{2}{nd}-\dfrac{1}{n^2}\right) \\ &\leq \sum_{d=1}^n \left( \dfrac{2}{nd}-\dfrac{1}{n^2}\right)\\ &\leq \dfrac{1}{n}+\dfrac{2}{n}\sum_{d=1}^n\dfrac{1}{d}=\dfrac{1}{n}+O\left( \dfrac{\log n}{n}\right)= O\left( \dfrac{\log n}{n}\right). \end{aligned}\]

  3. &

  4. La série \(\sum_d \mu(d)/d^2\) étant convergente \[\lim_{n\to\infty} r_n=\sum_{d=1}^{\infty}\dfrac{\mu(d)}{d^2},{(1)}\] d’un autre coté, nous pouvons écrire \[\begin{aligned}\dfrac{\pi^2}{6}\sum_{d=1}^{\infty}\dfrac{\mu(d)}{d^2} &=\left( \sum_{i=1}^\infty\dfrac{1}{i^2}\right) \left( \sum_{d=1}^{\infty}\dfrac{\mu(d)}{d^2}\right)\\ &=\sum_{(i,d)\in\mathbb N^2}\dfrac{\mu(d)}{i^2d^2} =\sum_{i\geq 1}\dfrac{1}{i^2}\sum_{l/i}\mu(l):= \sum_{i\geq 1}\dfrac{1}{i^2}S(i) \end{aligned}\] où les deux avant dernières égalités sont justifiées par l’absolue convergence des deux premieres séries (on peut alors sommer par paquets). il reste donc à estimer \(S(i)\) pour \(i\geq 1\). \(S(1)=1\) et pour \(i\geq 2\) soit \(i=q_1^{\alpha_1}\dots q_N^{\alpha_N}\) la décomposition de \(i\) en facteurs premiers ; un diviseur \(l\) de \(i\) s’écrit donc sous la forme \(l=q_1^{\beta_1}\dots q_N^{\beta_N}\) avec \(0\leq\beta_j\leq\alpha_j\). Mais \(\mu(l)\ne 0\) signifie que \(\beta_j=0\) ou \(1\) et dans ce cas \(\mu(l)=(-1)^s\)\(s=\text{card}\{1\leq j\leq N\ :\ \beta_j=1\}\). Il existe donc une bijection entre l’ensemble des diviseurs \(l\) de \(i\) tels que \(\mu(l)=(-1)^s\) et l’ensemble des \(N\)-uplets \((\beta_1,\dots,\beta_N)\in\{0,1\}^N\) avec exactement \(s\) composantes égales à \(1\) et ce dernier est bien entendu de cardinal \(C_N^s\) si bien que \[S(i)=\sum_{l/i}\mu(l)=\sum_{s=0}^N\sum_{l/i \& \mu(l)=(-1)^s}\mu(l)=\sum_{s=0}^N C_N^s(-1)^s=(1-1)^N=0.\] en résumé \[S(i)=\begin{cases} 1&\text{si}\quad i=1\\ 0&\text{si}\quad i\geq 2. \end{cases}\] soit \[\dfrac{\pi^2}{6}\sum_{d=1}^{\infty}\dfrac{\mu(d)}{d^2}=1\] et enfin \[\lim_{n\to\infty}r_n=\sum_{d=1}^{\infty}\dfrac{\mu(d)}{d^2}=\dfrac{6}{\pi^2}.\]


Documents à télécharger

Probabilité que deux entiers soient premiers entre-eux
Télécharger Télécharger avec les solutions et commentaires

L'exercice